Gravity magic - How does it work?












12












$begingroup$


This question graduated from the Sandbox for Proposed Questions





Gravity magic - How does it work?



(What physical law can magic manipulate in order to affect gravity?)



In my world everyone is capable of casting magic. It is possible for a single person to cast as many different types of magic as he/she wishes, assuming they know the spells. Every person also has an affinity to a specific type/form of magic, e.g. projectile magic, water magic or defensive magic. One of the main characters in my story has an affinity to gravity-based spells, so it is much easier and more intuitive to use them for her.



What I'm interested in is not how magic works in this context (assume there is some amount of recharging MP within every person that is used to cast spells), but rather how the gravitational effects caused by the magic could be explained.



I'm looking for a "generic" solution that explains explicitly this type of magic, not every single spell/usage, but rather "gravity-type magic" itself. I don't intend to explain my readers every single law of physics, space-time etc. in every detail, but the main aspect of "how" should come across.



When answering, take the following scenarios as use-cases for the magic to check:




  • Slowing down yourself down when falling, so you don't hit the ground at full speed (as near to 0m/s as possible, at the very least you shouldn't be injured by landing) (e.g. jumping from higher up)

  • Levitating yourself / other objects, so they don't move in any direction


    • Bonus points: still won't move if influenced by another force (e.g. a thrown stone)



  • Pushing objects out of your way (e.g. a locked door)

  • Ability to combine multiple "gravity fields", so you could simultaneously keep a cup in place and rotate it upwards-down, while still keeping the liquid inside


I've also found the question How do I grant superheroes gravitational manipulation, justified scientifically?, but it is somewhat different than my question. My question focuses on how to actually perform the gravity-based magic and not on how you could justify the existence of such powers.










share|improve this question









$endgroup$








  • 6




    $begingroup$
    Good job using the sandbox.
    $endgroup$
    – L.Dutch
    16 hours ago






  • 1




    $begingroup$
    Do note that changing your mass doesn't make you fall any slower unless you also change your buoyancy in air, so magic is in the air! Allez, allez, allez...
    $endgroup$
    – user6760
    12 hours ago








  • 3




    $begingroup$
    Brandon Sanderson's "The Stormlight Archive" series present a nice example of gravity magic. Aside from being a very good book, it may inspire you.
    $endgroup$
    – Echox
    8 hours ago






  • 2




    $begingroup$
    @Echox Come for the gravity magic, stay for the bizarrely alien worldbuilding, the setting that manages to make huge fantasy swords and magic power armor actually work, the spren, Shadesmar, the secret societies, the Unmade, the dozens of POV characters, the political intrigue, Wit, the personal drama... take your pick. Stormlight has something for everyone!
    $endgroup$
    – Mason Wheeler
    7 hours ago


















12












$begingroup$


This question graduated from the Sandbox for Proposed Questions





Gravity magic - How does it work?



(What physical law can magic manipulate in order to affect gravity?)



In my world everyone is capable of casting magic. It is possible for a single person to cast as many different types of magic as he/she wishes, assuming they know the spells. Every person also has an affinity to a specific type/form of magic, e.g. projectile magic, water magic or defensive magic. One of the main characters in my story has an affinity to gravity-based spells, so it is much easier and more intuitive to use them for her.



What I'm interested in is not how magic works in this context (assume there is some amount of recharging MP within every person that is used to cast spells), but rather how the gravitational effects caused by the magic could be explained.



I'm looking for a "generic" solution that explains explicitly this type of magic, not every single spell/usage, but rather "gravity-type magic" itself. I don't intend to explain my readers every single law of physics, space-time etc. in every detail, but the main aspect of "how" should come across.



When answering, take the following scenarios as use-cases for the magic to check:




  • Slowing down yourself down when falling, so you don't hit the ground at full speed (as near to 0m/s as possible, at the very least you shouldn't be injured by landing) (e.g. jumping from higher up)

  • Levitating yourself / other objects, so they don't move in any direction


    • Bonus points: still won't move if influenced by another force (e.g. a thrown stone)



  • Pushing objects out of your way (e.g. a locked door)

  • Ability to combine multiple "gravity fields", so you could simultaneously keep a cup in place and rotate it upwards-down, while still keeping the liquid inside


I've also found the question How do I grant superheroes gravitational manipulation, justified scientifically?, but it is somewhat different than my question. My question focuses on how to actually perform the gravity-based magic and not on how you could justify the existence of such powers.










share|improve this question









$endgroup$








  • 6




    $begingroup$
    Good job using the sandbox.
    $endgroup$
    – L.Dutch
    16 hours ago






  • 1




    $begingroup$
    Do note that changing your mass doesn't make you fall any slower unless you also change your buoyancy in air, so magic is in the air! Allez, allez, allez...
    $endgroup$
    – user6760
    12 hours ago








  • 3




    $begingroup$
    Brandon Sanderson's "The Stormlight Archive" series present a nice example of gravity magic. Aside from being a very good book, it may inspire you.
    $endgroup$
    – Echox
    8 hours ago






  • 2




    $begingroup$
    @Echox Come for the gravity magic, stay for the bizarrely alien worldbuilding, the setting that manages to make huge fantasy swords and magic power armor actually work, the spren, Shadesmar, the secret societies, the Unmade, the dozens of POV characters, the political intrigue, Wit, the personal drama... take your pick. Stormlight has something for everyone!
    $endgroup$
    – Mason Wheeler
    7 hours ago
















12












12








12


1



$begingroup$


This question graduated from the Sandbox for Proposed Questions





Gravity magic - How does it work?



(What physical law can magic manipulate in order to affect gravity?)



In my world everyone is capable of casting magic. It is possible for a single person to cast as many different types of magic as he/she wishes, assuming they know the spells. Every person also has an affinity to a specific type/form of magic, e.g. projectile magic, water magic or defensive magic. One of the main characters in my story has an affinity to gravity-based spells, so it is much easier and more intuitive to use them for her.



What I'm interested in is not how magic works in this context (assume there is some amount of recharging MP within every person that is used to cast spells), but rather how the gravitational effects caused by the magic could be explained.



I'm looking for a "generic" solution that explains explicitly this type of magic, not every single spell/usage, but rather "gravity-type magic" itself. I don't intend to explain my readers every single law of physics, space-time etc. in every detail, but the main aspect of "how" should come across.



When answering, take the following scenarios as use-cases for the magic to check:




  • Slowing down yourself down when falling, so you don't hit the ground at full speed (as near to 0m/s as possible, at the very least you shouldn't be injured by landing) (e.g. jumping from higher up)

  • Levitating yourself / other objects, so they don't move in any direction


    • Bonus points: still won't move if influenced by another force (e.g. a thrown stone)



  • Pushing objects out of your way (e.g. a locked door)

  • Ability to combine multiple "gravity fields", so you could simultaneously keep a cup in place and rotate it upwards-down, while still keeping the liquid inside


I've also found the question How do I grant superheroes gravitational manipulation, justified scientifically?, but it is somewhat different than my question. My question focuses on how to actually perform the gravity-based magic and not on how you could justify the existence of such powers.










share|improve this question









$endgroup$




This question graduated from the Sandbox for Proposed Questions





Gravity magic - How does it work?



(What physical law can magic manipulate in order to affect gravity?)



In my world everyone is capable of casting magic. It is possible for a single person to cast as many different types of magic as he/she wishes, assuming they know the spells. Every person also has an affinity to a specific type/form of magic, e.g. projectile magic, water magic or defensive magic. One of the main characters in my story has an affinity to gravity-based spells, so it is much easier and more intuitive to use them for her.



What I'm interested in is not how magic works in this context (assume there is some amount of recharging MP within every person that is used to cast spells), but rather how the gravitational effects caused by the magic could be explained.



I'm looking for a "generic" solution that explains explicitly this type of magic, not every single spell/usage, but rather "gravity-type magic" itself. I don't intend to explain my readers every single law of physics, space-time etc. in every detail, but the main aspect of "how" should come across.



When answering, take the following scenarios as use-cases for the magic to check:




  • Slowing down yourself down when falling, so you don't hit the ground at full speed (as near to 0m/s as possible, at the very least you shouldn't be injured by landing) (e.g. jumping from higher up)

  • Levitating yourself / other objects, so they don't move in any direction


    • Bonus points: still won't move if influenced by another force (e.g. a thrown stone)



  • Pushing objects out of your way (e.g. a locked door)

  • Ability to combine multiple "gravity fields", so you could simultaneously keep a cup in place and rotate it upwards-down, while still keeping the liquid inside


I've also found the question How do I grant superheroes gravitational manipulation, justified scientifically?, but it is somewhat different than my question. My question focuses on how to actually perform the gravity-based magic and not on how you could justify the existence of such powers.







science-based magic gravity






share|improve this question













share|improve this question











share|improve this question




share|improve this question










asked 16 hours ago









Tobias F.Tobias F.

1617




1617








  • 6




    $begingroup$
    Good job using the sandbox.
    $endgroup$
    – L.Dutch
    16 hours ago






  • 1




    $begingroup$
    Do note that changing your mass doesn't make you fall any slower unless you also change your buoyancy in air, so magic is in the air! Allez, allez, allez...
    $endgroup$
    – user6760
    12 hours ago








  • 3




    $begingroup$
    Brandon Sanderson's "The Stormlight Archive" series present a nice example of gravity magic. Aside from being a very good book, it may inspire you.
    $endgroup$
    – Echox
    8 hours ago






  • 2




    $begingroup$
    @Echox Come for the gravity magic, stay for the bizarrely alien worldbuilding, the setting that manages to make huge fantasy swords and magic power armor actually work, the spren, Shadesmar, the secret societies, the Unmade, the dozens of POV characters, the political intrigue, Wit, the personal drama... take your pick. Stormlight has something for everyone!
    $endgroup$
    – Mason Wheeler
    7 hours ago
















  • 6




    $begingroup$
    Good job using the sandbox.
    $endgroup$
    – L.Dutch
    16 hours ago






  • 1




    $begingroup$
    Do note that changing your mass doesn't make you fall any slower unless you also change your buoyancy in air, so magic is in the air! Allez, allez, allez...
    $endgroup$
    – user6760
    12 hours ago








  • 3




    $begingroup$
    Brandon Sanderson's "The Stormlight Archive" series present a nice example of gravity magic. Aside from being a very good book, it may inspire you.
    $endgroup$
    – Echox
    8 hours ago






  • 2




    $begingroup$
    @Echox Come for the gravity magic, stay for the bizarrely alien worldbuilding, the setting that manages to make huge fantasy swords and magic power armor actually work, the spren, Shadesmar, the secret societies, the Unmade, the dozens of POV characters, the political intrigue, Wit, the personal drama... take your pick. Stormlight has something for everyone!
    $endgroup$
    – Mason Wheeler
    7 hours ago










6




6




$begingroup$
Good job using the sandbox.
$endgroup$
– L.Dutch
16 hours ago




$begingroup$
Good job using the sandbox.
$endgroup$
– L.Dutch
16 hours ago




1




1




$begingroup$
Do note that changing your mass doesn't make you fall any slower unless you also change your buoyancy in air, so magic is in the air! Allez, allez, allez...
$endgroup$
– user6760
12 hours ago






$begingroup$
Do note that changing your mass doesn't make you fall any slower unless you also change your buoyancy in air, so magic is in the air! Allez, allez, allez...
$endgroup$
– user6760
12 hours ago






3




3




$begingroup$
Brandon Sanderson's "The Stormlight Archive" series present a nice example of gravity magic. Aside from being a very good book, it may inspire you.
$endgroup$
– Echox
8 hours ago




$begingroup$
Brandon Sanderson's "The Stormlight Archive" series present a nice example of gravity magic. Aside from being a very good book, it may inspire you.
$endgroup$
– Echox
8 hours ago




2




2




$begingroup$
@Echox Come for the gravity magic, stay for the bizarrely alien worldbuilding, the setting that manages to make huge fantasy swords and magic power armor actually work, the spren, Shadesmar, the secret societies, the Unmade, the dozens of POV characters, the political intrigue, Wit, the personal drama... take your pick. Stormlight has something for everyone!
$endgroup$
– Mason Wheeler
7 hours ago






$begingroup$
@Echox Come for the gravity magic, stay for the bizarrely alien worldbuilding, the setting that manages to make huge fantasy swords and magic power armor actually work, the spren, Shadesmar, the secret societies, the Unmade, the dozens of POV characters, the political intrigue, Wit, the personal drama... take your pick. Stormlight has something for everyone!
$endgroup$
– Mason Wheeler
7 hours ago












7 Answers
7






active

oldest

votes


















6












$begingroup$


What I'm interested in is not how magic works in this context (assume there is some amount of recharging MP within every person that is used to cast spells), but rather how the gravitational effects caused by the magic could be explained.




If you stick to Einstein's model for gravity, you can explain your magic via the ability of manipulating the curvature of space-time.



The Gravitational force experienced by an object (the Einstein tensor) subject to curvature $R$ (the Ricci tensor) can be calculated by the formula $G_{µv} = $$8πG over C^4$$ T_{µv}$



To create a "repulsion" effect, the curvature can be made convex instead of concave. To limit how large an area is affected, use a sharper curve (this will cause the force experience to increase / decrease more rapidly as you approach the peak, instead of the normal $r^{-2}$ relationship experienced at distance $r$ under standard conditions)



By curving space-time directly, you can also apply the force in any direction instead of just "towards" and "away"






share|improve this answer









$endgroup$





















    5












    $begingroup$


    What I'm interested in is not how magic works in this context (assume there is some amount of recharging MP within every person that is used to cast spells), but rather how the gravitational effects caused by the magic could be explained.




    If you stitch to the Newtonian model for gravity, you can explain your magic via the ability of controlling magnitude and sign of the masses involved in the gravitational interaction.



    The gravitational force between two bodies of mass m at distance r can be calculated according to the famous formula $F=G cdot$$m_1 cdot m_2 over r^2$.



    If you can change the sign of m, you can have gravity be repulsive. So, in case you jump from a skyscraper, turning your mass to negative once in fall would have the effect of Earth pushing you up, slowing down your fall. You can also levitate by alternating positive and negative mass values.



    Turning upside down a cup while not dropping its content could be achieved in the same way.






    share|improve this answer









    $endgroup$









    • 1




      $begingroup$
      One thing to consider is that gravity is ridiculously weak compared to any other force. So if you limit the amount you can change the magnitude and sign, you will likely only be able to either crush things or levitate them, if the effect is uniform, since you are always pushing against the earth. Nonuniform gravity can induce some spin in objects, but it will be really hard to generate any sideways motion
      $endgroup$
      – Whitecold
      15 hours ago





















    3












    $begingroup$

    Personally I'd steer clear of trying to get some physics-based in-universe explanation unless your magical society also has a very high technology.



    That said...



    Since gravity is the result of curvature of space, the simple answer would be to have the magic operate to alter that curvature. We don't know exactly what mechanism causes space to curve in the presence of mass, so you can get away with some artistic licence. It's magic after all.



    Let's say that the gravitational field strength, which is an expression of the warping of space, is something that you can move around. You can't make much gravity on your own, but you can potentially take energy from other parts of the field and concentrate it. Reducing the gravity in an area requires you to spread that field energy around, so gravity gets stronger elsewhere. The better you are at controlling it the more complex you can make the resulting changes, and the stronger your magical talent the larger the scope of the changes.



    But gravity only really does one thing: pull on stuff. You can put a gravity well in the air 2 feet above the table and watch it fall in, but controlling the orientation of the table is going to be difficult. You'd probably need to create a shaped gravity field the shape of the table to get it in the right orientation, and it's not just going to sit there when you push on it... unless you have enough field strength to counter the push.



    Since this is magic though, let's assume that you can deform space the way you want if you have the right spell. You could do all sorts of nasty things with it, like concentrating all of the gravity in a battlefield into a tiny ball of hyper-gravity strong enough to tear humans apart. You could make things fall in any direction, essentially making you the ultimate siege weapon. Water flowing uphill is way simple. How about castles falling into the sky? If it were my castle I'd be doing my best to outlaw gravity magic, as aggressively as possible.



    So yes, you can stop people falling. Yes, you can walk on walls and ceilings. But it's going to take a hell of a lot of field shaping to get cups to dance and flip without spilling their contents.



    Perhaps what you need is less gravity and more telekinesis? Directly applying forces to objects has a lot of potential too. Maybe a combination of both?






    share|improve this answer









    $endgroup$













    • $begingroup$
      We don't know, +1
      $endgroup$
      – Mazura
      25 mins ago



















    1












    $begingroup$

    Gravity sources are everywhere. Everything in the galaxy is attracting everything else.

    The only reason the satellites in space don't come crashing down is because they are traveling fast enough to keep missing the Earth.

    The only reason the Earth doesn't crash into the sun is that it's moving fast enough to keep missing it.

    The only reason the sun doesn't fall into the black hole at the galactic core is that it's moving fast enough to keep missing it.



    If any of them were to lose forward momentum they'd plunge to a fiery death.



    The only reason two apples don't orbit each other is because they are overwhelmed by earths gravity, and the suns gravity, and all of the other gravity sources that influence us.



    So the way that magic controls gravity is by selectively ignoring, reversing, or reinforcing it around whatever thing you want to manipulate. If you need to levitate, reverse the pull of Earths gravity. If you need to push something, reinforce the pull from things in line with the direction you want it to go, and then negate or reverse the pull from the other directions.



    At the highest levels it would be crazy powerful, being able to increase an items gravity field to the point where it becomes neutronium or a singularity.






    share|improve this answer









    $endgroup$





















      0












      $begingroup$

      This is more of an addition to L.Dutch♦'s answer, controlling magnitude and sign of the masses involved in the gravitational interaction, but to achieve some of the points you mentioned (pushing a door out of the way, stopping movement even when affected by an outside force) you need to have a solution for sideways motion and rotation.



      For sideways motion you can take into account the fact that Earth isn't a point source for gravity, instead we are pulled towards each particle that makes up the Earth and our gravity vector is the sum of all those forces. If your character can control the effect of gravity from different sources separately, then sideways motion can be achieved by getting a push from one half of the Earth and getting pulled by the other. You could even use gravity from other celestial bodies (Sun, Jupiter, supermassive black hole at the center of the the Milky Way).



      Rotation can be achieved as described in Whitecold's comment, with nonuniform gravity: have half of the thing you're rotating be pulled down and the other half pulled up (gently so you don't tear it apart). Your rotating cup of water that doesn't spill could be done by having gravity push "into" the cup just near the mouth, pushing from the bottom of the cup to counteract the movement the first push would impart, all the while keeping the cup and water in the air (this might require more skill from your character than simple levitation).






      share|improve this answer








      New contributor




      JCP is a new contributor to this site. Take care in asking for clarification, commenting, and answering.
      Check out our Code of Conduct.






      $endgroup$





















        0












        $begingroup$

        The Higgs field gives elementary particles their mass, and having some sort of a mechanism to attenuate or disable its interactions could work to your benefit at a level that doesn't really have a reasonably obvious counterargument. So in effect, the magic could somehow stop this interaction, making object momentarily more or less massive. (The remote pushing effect you mentioned I found the most difficult to explain...) The intricacies work very much differently for charged leptons and force carrier bosons, though.



        Most of the mass of matter is from the mass of the atomic nuclei, and ≈99% of that mass doesn't come from the mass of the individual elementary particles but the strong force; potential energies of the quarks (and hence protons and neutrons) bound together can be translated to mass via E=mc^2. Another explanation for your magic could thus be not exactly one that works by meddling with gravity but meddling with another fundamental force which affects mass which in turn gets picked up unmodified gravity. The strong force is mediated by gluons (analogous to photons for the electromagnetic interactions), so off we go with gluemancy.



        The problem with this all is of course, that if one is capable enough to modify these tine, fundamental properties of particles, it doesn't easily make much sense to be able to only demonstrate gravity-based powers.






        share|improve this answer








        New contributor




        JoonasD6 is a new contributor to this site. Take care in asking for clarification, commenting, and answering.
        Check out our Code of Conduct.






        $endgroup$





















          0












          $begingroup$

          The kind of magic you are talking about is anthropomorphic.





          • Slowing down yourself down when falling, so you don't hit the ground at full speed (as near to 0m/s as possible, at the very least you shouldn't be injured by landing) (e.g. jumping from higher up)

          • Levitating yourself / other objects, so they don't move in any direction

          • Bonus points: still won't move if influenced by another force (e.g. a thrown stone)

          • Pushing objects out of your way (e.g. a locked door)

          • Ability to combine multiple "gravity fields", so you could simultaneously keep a cup in place and rotate it upwards-down, while still keeping the liquid inside




          Gravity is the measurement of the degree to which mass-energy curves space time around it. What you describe above is a bunch of stuff that seems convincing to a human if you described gravity magic to.



          A physics based process won't neatly fit in a "stuff that X magic would do" that a human would narrate, because physics doesn't really care what a human finds narratively convincing.



          As an example, Electro-Magnitism covers everything from being able to touch things, see things, shoot lightning, and pretty much all of chemistry. Meanwhile, the human "story" based Magnatism is about being able to move metals around (because they are "magnetic"), and Electricty magic is about lightning bolts and static electricty.



          If you want anthropomorphic magic, magic that makes sense mythologically or in categories humans care about, it has to be powered by an intelligence with a similar mind to ours. That intelligence somehow set up the rules in ways that we find are pleasing.



          Spells become "instructions" to this intelligence, who then does what asked. The being being interacted with could be a god-like entity, a demon, or a post-singularity AI, or an ancient forgotten society of super-scientists who left "macros" active that their children used to play with.



          In the last case, the affinity of certain people for certain kinds of magic is akin to being able to unlock a certain smart phone with your voice, because it coincidentally is similar to the real owner (it doesn't have to be voice-locked, but you can imagine it being so). The fact that these ancient intelligences let their children play with fireballs would be explained by the fact that everyone in their society was backed up, and repair/resurrection was trivial; that technology doesn't recognize humans as being in their pattern-banks. Humans who can heal are actually using the "veternarian" macros.



          Now, once we divorce the physical mechanism from the "kind of magic", the fact that these are "gravity" macros is because they are the kind of thing that the parent programmed for their kid (or maybe bought the "play with gravity" kit for their kid's system).



          Your entire environment could be a simulation running in an ancient computer, with the original hosts long exinct. The matter you are walking on and breathing could be programmable computronium (mass-energy optimized to do computation) that emulates being matter so as to leave a "natural" experience for growing up, but being computronium this is just an option; "magic" literally ignores the apparent laws of physics, because the laws of physics as you experience them are just an emulation layer.



          Being able to float in either of these cases is little more than a matter of changing the position value of a variable in a specific coordinate space. Now, no human has access to this underlying system; so they cannot insert the wrong arguments and cause the water on the earth to start to orbit the other way around the sun.






          share|improve this answer









          $endgroup$













            Your Answer





            StackExchange.ifUsing("editor", function () {
            return StackExchange.using("mathjaxEditing", function () {
            StackExchange.MarkdownEditor.creationCallbacks.add(function (editor, postfix) {
            StackExchange.mathjaxEditing.prepareWmdForMathJax(editor, postfix, [["$", "$"], ["\\(","\\)"]]);
            });
            });
            }, "mathjax-editing");

            StackExchange.ready(function() {
            var channelOptions = {
            tags: "".split(" "),
            id: "579"
            };
            initTagRenderer("".split(" "), "".split(" "), channelOptions);

            StackExchange.using("externalEditor", function() {
            // Have to fire editor after snippets, if snippets enabled
            if (StackExchange.settings.snippets.snippetsEnabled) {
            StackExchange.using("snippets", function() {
            createEditor();
            });
            }
            else {
            createEditor();
            }
            });

            function createEditor() {
            StackExchange.prepareEditor({
            heartbeatType: 'answer',
            autoActivateHeartbeat: false,
            convertImagesToLinks: false,
            noModals: true,
            showLowRepImageUploadWarning: true,
            reputationToPostImages: null,
            bindNavPrevention: true,
            postfix: "",
            imageUploader: {
            brandingHtml: "Powered by u003ca class="icon-imgur-white" href="https://imgur.com/"u003eu003c/au003e",
            contentPolicyHtml: "User contributions licensed under u003ca href="https://creativecommons.org/licenses/by-sa/3.0/"u003ecc by-sa 3.0 with attribution requiredu003c/au003e u003ca href="https://stackoverflow.com/legal/content-policy"u003e(content policy)u003c/au003e",
            allowUrls: true
            },
            noCode: true, onDemand: true,
            discardSelector: ".discard-answer"
            ,immediatelyShowMarkdownHelp:true
            });


            }
            });














            draft saved

            draft discarded


















            StackExchange.ready(
            function () {
            StackExchange.openid.initPostLogin('.new-post-login', 'https%3a%2f%2fworldbuilding.stackexchange.com%2fquestions%2f141533%2fgravity-magic-how-does-it-work%23new-answer', 'question_page');
            }
            );

            Post as a guest















            Required, but never shown

























            7 Answers
            7






            active

            oldest

            votes








            7 Answers
            7






            active

            oldest

            votes









            active

            oldest

            votes






            active

            oldest

            votes









            6












            $begingroup$


            What I'm interested in is not how magic works in this context (assume there is some amount of recharging MP within every person that is used to cast spells), but rather how the gravitational effects caused by the magic could be explained.




            If you stick to Einstein's model for gravity, you can explain your magic via the ability of manipulating the curvature of space-time.



            The Gravitational force experienced by an object (the Einstein tensor) subject to curvature $R$ (the Ricci tensor) can be calculated by the formula $G_{µv} = $$8πG over C^4$$ T_{µv}$



            To create a "repulsion" effect, the curvature can be made convex instead of concave. To limit how large an area is affected, use a sharper curve (this will cause the force experience to increase / decrease more rapidly as you approach the peak, instead of the normal $r^{-2}$ relationship experienced at distance $r$ under standard conditions)



            By curving space-time directly, you can also apply the force in any direction instead of just "towards" and "away"






            share|improve this answer









            $endgroup$


















              6












              $begingroup$


              What I'm interested in is not how magic works in this context (assume there is some amount of recharging MP within every person that is used to cast spells), but rather how the gravitational effects caused by the magic could be explained.




              If you stick to Einstein's model for gravity, you can explain your magic via the ability of manipulating the curvature of space-time.



              The Gravitational force experienced by an object (the Einstein tensor) subject to curvature $R$ (the Ricci tensor) can be calculated by the formula $G_{µv} = $$8πG over C^4$$ T_{µv}$



              To create a "repulsion" effect, the curvature can be made convex instead of concave. To limit how large an area is affected, use a sharper curve (this will cause the force experience to increase / decrease more rapidly as you approach the peak, instead of the normal $r^{-2}$ relationship experienced at distance $r$ under standard conditions)



              By curving space-time directly, you can also apply the force in any direction instead of just "towards" and "away"






              share|improve this answer









              $endgroup$
















                6












                6








                6





                $begingroup$


                What I'm interested in is not how magic works in this context (assume there is some amount of recharging MP within every person that is used to cast spells), but rather how the gravitational effects caused by the magic could be explained.




                If you stick to Einstein's model for gravity, you can explain your magic via the ability of manipulating the curvature of space-time.



                The Gravitational force experienced by an object (the Einstein tensor) subject to curvature $R$ (the Ricci tensor) can be calculated by the formula $G_{µv} = $$8πG over C^4$$ T_{µv}$



                To create a "repulsion" effect, the curvature can be made convex instead of concave. To limit how large an area is affected, use a sharper curve (this will cause the force experience to increase / decrease more rapidly as you approach the peak, instead of the normal $r^{-2}$ relationship experienced at distance $r$ under standard conditions)



                By curving space-time directly, you can also apply the force in any direction instead of just "towards" and "away"






                share|improve this answer









                $endgroup$




                What I'm interested in is not how magic works in this context (assume there is some amount of recharging MP within every person that is used to cast spells), but rather how the gravitational effects caused by the magic could be explained.




                If you stick to Einstein's model for gravity, you can explain your magic via the ability of manipulating the curvature of space-time.



                The Gravitational force experienced by an object (the Einstein tensor) subject to curvature $R$ (the Ricci tensor) can be calculated by the formula $G_{µv} = $$8πG over C^4$$ T_{µv}$



                To create a "repulsion" effect, the curvature can be made convex instead of concave. To limit how large an area is affected, use a sharper curve (this will cause the force experience to increase / decrease more rapidly as you approach the peak, instead of the normal $r^{-2}$ relationship experienced at distance $r$ under standard conditions)



                By curving space-time directly, you can also apply the force in any direction instead of just "towards" and "away"







                share|improve this answer












                share|improve this answer



                share|improve this answer










                answered 12 hours ago









                ChronocidalChronocidal

                6,4031831




                6,4031831























                    5












                    $begingroup$


                    What I'm interested in is not how magic works in this context (assume there is some amount of recharging MP within every person that is used to cast spells), but rather how the gravitational effects caused by the magic could be explained.




                    If you stitch to the Newtonian model for gravity, you can explain your magic via the ability of controlling magnitude and sign of the masses involved in the gravitational interaction.



                    The gravitational force between two bodies of mass m at distance r can be calculated according to the famous formula $F=G cdot$$m_1 cdot m_2 over r^2$.



                    If you can change the sign of m, you can have gravity be repulsive. So, in case you jump from a skyscraper, turning your mass to negative once in fall would have the effect of Earth pushing you up, slowing down your fall. You can also levitate by alternating positive and negative mass values.



                    Turning upside down a cup while not dropping its content could be achieved in the same way.






                    share|improve this answer









                    $endgroup$









                    • 1




                      $begingroup$
                      One thing to consider is that gravity is ridiculously weak compared to any other force. So if you limit the amount you can change the magnitude and sign, you will likely only be able to either crush things or levitate them, if the effect is uniform, since you are always pushing against the earth. Nonuniform gravity can induce some spin in objects, but it will be really hard to generate any sideways motion
                      $endgroup$
                      – Whitecold
                      15 hours ago


















                    5












                    $begingroup$


                    What I'm interested in is not how magic works in this context (assume there is some amount of recharging MP within every person that is used to cast spells), but rather how the gravitational effects caused by the magic could be explained.




                    If you stitch to the Newtonian model for gravity, you can explain your magic via the ability of controlling magnitude and sign of the masses involved in the gravitational interaction.



                    The gravitational force between two bodies of mass m at distance r can be calculated according to the famous formula $F=G cdot$$m_1 cdot m_2 over r^2$.



                    If you can change the sign of m, you can have gravity be repulsive. So, in case you jump from a skyscraper, turning your mass to negative once in fall would have the effect of Earth pushing you up, slowing down your fall. You can also levitate by alternating positive and negative mass values.



                    Turning upside down a cup while not dropping its content could be achieved in the same way.






                    share|improve this answer









                    $endgroup$









                    • 1




                      $begingroup$
                      One thing to consider is that gravity is ridiculously weak compared to any other force. So if you limit the amount you can change the magnitude and sign, you will likely only be able to either crush things or levitate them, if the effect is uniform, since you are always pushing against the earth. Nonuniform gravity can induce some spin in objects, but it will be really hard to generate any sideways motion
                      $endgroup$
                      – Whitecold
                      15 hours ago
















                    5












                    5








                    5





                    $begingroup$


                    What I'm interested in is not how magic works in this context (assume there is some amount of recharging MP within every person that is used to cast spells), but rather how the gravitational effects caused by the magic could be explained.




                    If you stitch to the Newtonian model for gravity, you can explain your magic via the ability of controlling magnitude and sign of the masses involved in the gravitational interaction.



                    The gravitational force between two bodies of mass m at distance r can be calculated according to the famous formula $F=G cdot$$m_1 cdot m_2 over r^2$.



                    If you can change the sign of m, you can have gravity be repulsive. So, in case you jump from a skyscraper, turning your mass to negative once in fall would have the effect of Earth pushing you up, slowing down your fall. You can also levitate by alternating positive and negative mass values.



                    Turning upside down a cup while not dropping its content could be achieved in the same way.






                    share|improve this answer









                    $endgroup$




                    What I'm interested in is not how magic works in this context (assume there is some amount of recharging MP within every person that is used to cast spells), but rather how the gravitational effects caused by the magic could be explained.




                    If you stitch to the Newtonian model for gravity, you can explain your magic via the ability of controlling magnitude and sign of the masses involved in the gravitational interaction.



                    The gravitational force between two bodies of mass m at distance r can be calculated according to the famous formula $F=G cdot$$m_1 cdot m_2 over r^2$.



                    If you can change the sign of m, you can have gravity be repulsive. So, in case you jump from a skyscraper, turning your mass to negative once in fall would have the effect of Earth pushing you up, slowing down your fall. You can also levitate by alternating positive and negative mass values.



                    Turning upside down a cup while not dropping its content could be achieved in the same way.







                    share|improve this answer












                    share|improve this answer



                    share|improve this answer










                    answered 16 hours ago









                    L.DutchL.Dutch

                    87.6k29205427




                    87.6k29205427








                    • 1




                      $begingroup$
                      One thing to consider is that gravity is ridiculously weak compared to any other force. So if you limit the amount you can change the magnitude and sign, you will likely only be able to either crush things or levitate them, if the effect is uniform, since you are always pushing against the earth. Nonuniform gravity can induce some spin in objects, but it will be really hard to generate any sideways motion
                      $endgroup$
                      – Whitecold
                      15 hours ago
















                    • 1




                      $begingroup$
                      One thing to consider is that gravity is ridiculously weak compared to any other force. So if you limit the amount you can change the magnitude and sign, you will likely only be able to either crush things or levitate them, if the effect is uniform, since you are always pushing against the earth. Nonuniform gravity can induce some spin in objects, but it will be really hard to generate any sideways motion
                      $endgroup$
                      – Whitecold
                      15 hours ago










                    1




                    1




                    $begingroup$
                    One thing to consider is that gravity is ridiculously weak compared to any other force. So if you limit the amount you can change the magnitude and sign, you will likely only be able to either crush things or levitate them, if the effect is uniform, since you are always pushing against the earth. Nonuniform gravity can induce some spin in objects, but it will be really hard to generate any sideways motion
                    $endgroup$
                    – Whitecold
                    15 hours ago






                    $begingroup$
                    One thing to consider is that gravity is ridiculously weak compared to any other force. So if you limit the amount you can change the magnitude and sign, you will likely only be able to either crush things or levitate them, if the effect is uniform, since you are always pushing against the earth. Nonuniform gravity can induce some spin in objects, but it will be really hard to generate any sideways motion
                    $endgroup$
                    – Whitecold
                    15 hours ago













                    3












                    $begingroup$

                    Personally I'd steer clear of trying to get some physics-based in-universe explanation unless your magical society also has a very high technology.



                    That said...



                    Since gravity is the result of curvature of space, the simple answer would be to have the magic operate to alter that curvature. We don't know exactly what mechanism causes space to curve in the presence of mass, so you can get away with some artistic licence. It's magic after all.



                    Let's say that the gravitational field strength, which is an expression of the warping of space, is something that you can move around. You can't make much gravity on your own, but you can potentially take energy from other parts of the field and concentrate it. Reducing the gravity in an area requires you to spread that field energy around, so gravity gets stronger elsewhere. The better you are at controlling it the more complex you can make the resulting changes, and the stronger your magical talent the larger the scope of the changes.



                    But gravity only really does one thing: pull on stuff. You can put a gravity well in the air 2 feet above the table and watch it fall in, but controlling the orientation of the table is going to be difficult. You'd probably need to create a shaped gravity field the shape of the table to get it in the right orientation, and it's not just going to sit there when you push on it... unless you have enough field strength to counter the push.



                    Since this is magic though, let's assume that you can deform space the way you want if you have the right spell. You could do all sorts of nasty things with it, like concentrating all of the gravity in a battlefield into a tiny ball of hyper-gravity strong enough to tear humans apart. You could make things fall in any direction, essentially making you the ultimate siege weapon. Water flowing uphill is way simple. How about castles falling into the sky? If it were my castle I'd be doing my best to outlaw gravity magic, as aggressively as possible.



                    So yes, you can stop people falling. Yes, you can walk on walls and ceilings. But it's going to take a hell of a lot of field shaping to get cups to dance and flip without spilling their contents.



                    Perhaps what you need is less gravity and more telekinesis? Directly applying forces to objects has a lot of potential too. Maybe a combination of both?






                    share|improve this answer









                    $endgroup$













                    • $begingroup$
                      We don't know, +1
                      $endgroup$
                      – Mazura
                      25 mins ago
















                    3












                    $begingroup$

                    Personally I'd steer clear of trying to get some physics-based in-universe explanation unless your magical society also has a very high technology.



                    That said...



                    Since gravity is the result of curvature of space, the simple answer would be to have the magic operate to alter that curvature. We don't know exactly what mechanism causes space to curve in the presence of mass, so you can get away with some artistic licence. It's magic after all.



                    Let's say that the gravitational field strength, which is an expression of the warping of space, is something that you can move around. You can't make much gravity on your own, but you can potentially take energy from other parts of the field and concentrate it. Reducing the gravity in an area requires you to spread that field energy around, so gravity gets stronger elsewhere. The better you are at controlling it the more complex you can make the resulting changes, and the stronger your magical talent the larger the scope of the changes.



                    But gravity only really does one thing: pull on stuff. You can put a gravity well in the air 2 feet above the table and watch it fall in, but controlling the orientation of the table is going to be difficult. You'd probably need to create a shaped gravity field the shape of the table to get it in the right orientation, and it's not just going to sit there when you push on it... unless you have enough field strength to counter the push.



                    Since this is magic though, let's assume that you can deform space the way you want if you have the right spell. You could do all sorts of nasty things with it, like concentrating all of the gravity in a battlefield into a tiny ball of hyper-gravity strong enough to tear humans apart. You could make things fall in any direction, essentially making you the ultimate siege weapon. Water flowing uphill is way simple. How about castles falling into the sky? If it were my castle I'd be doing my best to outlaw gravity magic, as aggressively as possible.



                    So yes, you can stop people falling. Yes, you can walk on walls and ceilings. But it's going to take a hell of a lot of field shaping to get cups to dance and flip without spilling their contents.



                    Perhaps what you need is less gravity and more telekinesis? Directly applying forces to objects has a lot of potential too. Maybe a combination of both?






                    share|improve this answer









                    $endgroup$













                    • $begingroup$
                      We don't know, +1
                      $endgroup$
                      – Mazura
                      25 mins ago














                    3












                    3








                    3





                    $begingroup$

                    Personally I'd steer clear of trying to get some physics-based in-universe explanation unless your magical society also has a very high technology.



                    That said...



                    Since gravity is the result of curvature of space, the simple answer would be to have the magic operate to alter that curvature. We don't know exactly what mechanism causes space to curve in the presence of mass, so you can get away with some artistic licence. It's magic after all.



                    Let's say that the gravitational field strength, which is an expression of the warping of space, is something that you can move around. You can't make much gravity on your own, but you can potentially take energy from other parts of the field and concentrate it. Reducing the gravity in an area requires you to spread that field energy around, so gravity gets stronger elsewhere. The better you are at controlling it the more complex you can make the resulting changes, and the stronger your magical talent the larger the scope of the changes.



                    But gravity only really does one thing: pull on stuff. You can put a gravity well in the air 2 feet above the table and watch it fall in, but controlling the orientation of the table is going to be difficult. You'd probably need to create a shaped gravity field the shape of the table to get it in the right orientation, and it's not just going to sit there when you push on it... unless you have enough field strength to counter the push.



                    Since this is magic though, let's assume that you can deform space the way you want if you have the right spell. You could do all sorts of nasty things with it, like concentrating all of the gravity in a battlefield into a tiny ball of hyper-gravity strong enough to tear humans apart. You could make things fall in any direction, essentially making you the ultimate siege weapon. Water flowing uphill is way simple. How about castles falling into the sky? If it were my castle I'd be doing my best to outlaw gravity magic, as aggressively as possible.



                    So yes, you can stop people falling. Yes, you can walk on walls and ceilings. But it's going to take a hell of a lot of field shaping to get cups to dance and flip without spilling their contents.



                    Perhaps what you need is less gravity and more telekinesis? Directly applying forces to objects has a lot of potential too. Maybe a combination of both?






                    share|improve this answer









                    $endgroup$



                    Personally I'd steer clear of trying to get some physics-based in-universe explanation unless your magical society also has a very high technology.



                    That said...



                    Since gravity is the result of curvature of space, the simple answer would be to have the magic operate to alter that curvature. We don't know exactly what mechanism causes space to curve in the presence of mass, so you can get away with some artistic licence. It's magic after all.



                    Let's say that the gravitational field strength, which is an expression of the warping of space, is something that you can move around. You can't make much gravity on your own, but you can potentially take energy from other parts of the field and concentrate it. Reducing the gravity in an area requires you to spread that field energy around, so gravity gets stronger elsewhere. The better you are at controlling it the more complex you can make the resulting changes, and the stronger your magical talent the larger the scope of the changes.



                    But gravity only really does one thing: pull on stuff. You can put a gravity well in the air 2 feet above the table and watch it fall in, but controlling the orientation of the table is going to be difficult. You'd probably need to create a shaped gravity field the shape of the table to get it in the right orientation, and it's not just going to sit there when you push on it... unless you have enough field strength to counter the push.



                    Since this is magic though, let's assume that you can deform space the way you want if you have the right spell. You could do all sorts of nasty things with it, like concentrating all of the gravity in a battlefield into a tiny ball of hyper-gravity strong enough to tear humans apart. You could make things fall in any direction, essentially making you the ultimate siege weapon. Water flowing uphill is way simple. How about castles falling into the sky? If it were my castle I'd be doing my best to outlaw gravity magic, as aggressively as possible.



                    So yes, you can stop people falling. Yes, you can walk on walls and ceilings. But it's going to take a hell of a lot of field shaping to get cups to dance and flip without spilling their contents.



                    Perhaps what you need is less gravity and more telekinesis? Directly applying forces to objects has a lot of potential too. Maybe a combination of both?







                    share|improve this answer












                    share|improve this answer



                    share|improve this answer










                    answered 11 hours ago









                    CoreyCorey

                    924411




                    924411












                    • $begingroup$
                      We don't know, +1
                      $endgroup$
                      – Mazura
                      25 mins ago


















                    • $begingroup$
                      We don't know, +1
                      $endgroup$
                      – Mazura
                      25 mins ago
















                    $begingroup$
                    We don't know, +1
                    $endgroup$
                    – Mazura
                    25 mins ago




                    $begingroup$
                    We don't know, +1
                    $endgroup$
                    – Mazura
                    25 mins ago











                    1












                    $begingroup$

                    Gravity sources are everywhere. Everything in the galaxy is attracting everything else.

                    The only reason the satellites in space don't come crashing down is because they are traveling fast enough to keep missing the Earth.

                    The only reason the Earth doesn't crash into the sun is that it's moving fast enough to keep missing it.

                    The only reason the sun doesn't fall into the black hole at the galactic core is that it's moving fast enough to keep missing it.



                    If any of them were to lose forward momentum they'd plunge to a fiery death.



                    The only reason two apples don't orbit each other is because they are overwhelmed by earths gravity, and the suns gravity, and all of the other gravity sources that influence us.



                    So the way that magic controls gravity is by selectively ignoring, reversing, or reinforcing it around whatever thing you want to manipulate. If you need to levitate, reverse the pull of Earths gravity. If you need to push something, reinforce the pull from things in line with the direction you want it to go, and then negate or reverse the pull from the other directions.



                    At the highest levels it would be crazy powerful, being able to increase an items gravity field to the point where it becomes neutronium or a singularity.






                    share|improve this answer









                    $endgroup$


















                      1












                      $begingroup$

                      Gravity sources are everywhere. Everything in the galaxy is attracting everything else.

                      The only reason the satellites in space don't come crashing down is because they are traveling fast enough to keep missing the Earth.

                      The only reason the Earth doesn't crash into the sun is that it's moving fast enough to keep missing it.

                      The only reason the sun doesn't fall into the black hole at the galactic core is that it's moving fast enough to keep missing it.



                      If any of them were to lose forward momentum they'd plunge to a fiery death.



                      The only reason two apples don't orbit each other is because they are overwhelmed by earths gravity, and the suns gravity, and all of the other gravity sources that influence us.



                      So the way that magic controls gravity is by selectively ignoring, reversing, or reinforcing it around whatever thing you want to manipulate. If you need to levitate, reverse the pull of Earths gravity. If you need to push something, reinforce the pull from things in line with the direction you want it to go, and then negate or reverse the pull from the other directions.



                      At the highest levels it would be crazy powerful, being able to increase an items gravity field to the point where it becomes neutronium or a singularity.






                      share|improve this answer









                      $endgroup$
















                        1












                        1








                        1





                        $begingroup$

                        Gravity sources are everywhere. Everything in the galaxy is attracting everything else.

                        The only reason the satellites in space don't come crashing down is because they are traveling fast enough to keep missing the Earth.

                        The only reason the Earth doesn't crash into the sun is that it's moving fast enough to keep missing it.

                        The only reason the sun doesn't fall into the black hole at the galactic core is that it's moving fast enough to keep missing it.



                        If any of them were to lose forward momentum they'd plunge to a fiery death.



                        The only reason two apples don't orbit each other is because they are overwhelmed by earths gravity, and the suns gravity, and all of the other gravity sources that influence us.



                        So the way that magic controls gravity is by selectively ignoring, reversing, or reinforcing it around whatever thing you want to manipulate. If you need to levitate, reverse the pull of Earths gravity. If you need to push something, reinforce the pull from things in line with the direction you want it to go, and then negate or reverse the pull from the other directions.



                        At the highest levels it would be crazy powerful, being able to increase an items gravity field to the point where it becomes neutronium or a singularity.






                        share|improve this answer









                        $endgroup$



                        Gravity sources are everywhere. Everything in the galaxy is attracting everything else.

                        The only reason the satellites in space don't come crashing down is because they are traveling fast enough to keep missing the Earth.

                        The only reason the Earth doesn't crash into the sun is that it's moving fast enough to keep missing it.

                        The only reason the sun doesn't fall into the black hole at the galactic core is that it's moving fast enough to keep missing it.



                        If any of them were to lose forward momentum they'd plunge to a fiery death.



                        The only reason two apples don't orbit each other is because they are overwhelmed by earths gravity, and the suns gravity, and all of the other gravity sources that influence us.



                        So the way that magic controls gravity is by selectively ignoring, reversing, or reinforcing it around whatever thing you want to manipulate. If you need to levitate, reverse the pull of Earths gravity. If you need to push something, reinforce the pull from things in line with the direction you want it to go, and then negate or reverse the pull from the other directions.



                        At the highest levels it would be crazy powerful, being able to increase an items gravity field to the point where it becomes neutronium or a singularity.







                        share|improve this answer












                        share|improve this answer



                        share|improve this answer










                        answered 10 hours ago









                        AndyD273AndyD273

                        31.1k258137




                        31.1k258137























                            0












                            $begingroup$

                            This is more of an addition to L.Dutch♦'s answer, controlling magnitude and sign of the masses involved in the gravitational interaction, but to achieve some of the points you mentioned (pushing a door out of the way, stopping movement even when affected by an outside force) you need to have a solution for sideways motion and rotation.



                            For sideways motion you can take into account the fact that Earth isn't a point source for gravity, instead we are pulled towards each particle that makes up the Earth and our gravity vector is the sum of all those forces. If your character can control the effect of gravity from different sources separately, then sideways motion can be achieved by getting a push from one half of the Earth and getting pulled by the other. You could even use gravity from other celestial bodies (Sun, Jupiter, supermassive black hole at the center of the the Milky Way).



                            Rotation can be achieved as described in Whitecold's comment, with nonuniform gravity: have half of the thing you're rotating be pulled down and the other half pulled up (gently so you don't tear it apart). Your rotating cup of water that doesn't spill could be done by having gravity push "into" the cup just near the mouth, pushing from the bottom of the cup to counteract the movement the first push would impart, all the while keeping the cup and water in the air (this might require more skill from your character than simple levitation).






                            share|improve this answer








                            New contributor




                            JCP is a new contributor to this site. Take care in asking for clarification, commenting, and answering.
                            Check out our Code of Conduct.






                            $endgroup$


















                              0












                              $begingroup$

                              This is more of an addition to L.Dutch♦'s answer, controlling magnitude and sign of the masses involved in the gravitational interaction, but to achieve some of the points you mentioned (pushing a door out of the way, stopping movement even when affected by an outside force) you need to have a solution for sideways motion and rotation.



                              For sideways motion you can take into account the fact that Earth isn't a point source for gravity, instead we are pulled towards each particle that makes up the Earth and our gravity vector is the sum of all those forces. If your character can control the effect of gravity from different sources separately, then sideways motion can be achieved by getting a push from one half of the Earth and getting pulled by the other. You could even use gravity from other celestial bodies (Sun, Jupiter, supermassive black hole at the center of the the Milky Way).



                              Rotation can be achieved as described in Whitecold's comment, with nonuniform gravity: have half of the thing you're rotating be pulled down and the other half pulled up (gently so you don't tear it apart). Your rotating cup of water that doesn't spill could be done by having gravity push "into" the cup just near the mouth, pushing from the bottom of the cup to counteract the movement the first push would impart, all the while keeping the cup and water in the air (this might require more skill from your character than simple levitation).






                              share|improve this answer








                              New contributor




                              JCP is a new contributor to this site. Take care in asking for clarification, commenting, and answering.
                              Check out our Code of Conduct.






                              $endgroup$
















                                0












                                0








                                0





                                $begingroup$

                                This is more of an addition to L.Dutch♦'s answer, controlling magnitude and sign of the masses involved in the gravitational interaction, but to achieve some of the points you mentioned (pushing a door out of the way, stopping movement even when affected by an outside force) you need to have a solution for sideways motion and rotation.



                                For sideways motion you can take into account the fact that Earth isn't a point source for gravity, instead we are pulled towards each particle that makes up the Earth and our gravity vector is the sum of all those forces. If your character can control the effect of gravity from different sources separately, then sideways motion can be achieved by getting a push from one half of the Earth and getting pulled by the other. You could even use gravity from other celestial bodies (Sun, Jupiter, supermassive black hole at the center of the the Milky Way).



                                Rotation can be achieved as described in Whitecold's comment, with nonuniform gravity: have half of the thing you're rotating be pulled down and the other half pulled up (gently so you don't tear it apart). Your rotating cup of water that doesn't spill could be done by having gravity push "into" the cup just near the mouth, pushing from the bottom of the cup to counteract the movement the first push would impart, all the while keeping the cup and water in the air (this might require more skill from your character than simple levitation).






                                share|improve this answer








                                New contributor




                                JCP is a new contributor to this site. Take care in asking for clarification, commenting, and answering.
                                Check out our Code of Conduct.






                                $endgroup$



                                This is more of an addition to L.Dutch♦'s answer, controlling magnitude and sign of the masses involved in the gravitational interaction, but to achieve some of the points you mentioned (pushing a door out of the way, stopping movement even when affected by an outside force) you need to have a solution for sideways motion and rotation.



                                For sideways motion you can take into account the fact that Earth isn't a point source for gravity, instead we are pulled towards each particle that makes up the Earth and our gravity vector is the sum of all those forces. If your character can control the effect of gravity from different sources separately, then sideways motion can be achieved by getting a push from one half of the Earth and getting pulled by the other. You could even use gravity from other celestial bodies (Sun, Jupiter, supermassive black hole at the center of the the Milky Way).



                                Rotation can be achieved as described in Whitecold's comment, with nonuniform gravity: have half of the thing you're rotating be pulled down and the other half pulled up (gently so you don't tear it apart). Your rotating cup of water that doesn't spill could be done by having gravity push "into" the cup just near the mouth, pushing from the bottom of the cup to counteract the movement the first push would impart, all the while keeping the cup and water in the air (this might require more skill from your character than simple levitation).







                                share|improve this answer








                                New contributor




                                JCP is a new contributor to this site. Take care in asking for clarification, commenting, and answering.
                                Check out our Code of Conduct.









                                share|improve this answer



                                share|improve this answer






                                New contributor




                                JCP is a new contributor to this site. Take care in asking for clarification, commenting, and answering.
                                Check out our Code of Conduct.









                                answered 11 hours ago









                                JCPJCP

                                1




                                1




                                New contributor




                                JCP is a new contributor to this site. Take care in asking for clarification, commenting, and answering.
                                Check out our Code of Conduct.





                                New contributor





                                JCP is a new contributor to this site. Take care in asking for clarification, commenting, and answering.
                                Check out our Code of Conduct.






                                JCP is a new contributor to this site. Take care in asking for clarification, commenting, and answering.
                                Check out our Code of Conduct.























                                    0












                                    $begingroup$

                                    The Higgs field gives elementary particles their mass, and having some sort of a mechanism to attenuate or disable its interactions could work to your benefit at a level that doesn't really have a reasonably obvious counterargument. So in effect, the magic could somehow stop this interaction, making object momentarily more or less massive. (The remote pushing effect you mentioned I found the most difficult to explain...) The intricacies work very much differently for charged leptons and force carrier bosons, though.



                                    Most of the mass of matter is from the mass of the atomic nuclei, and ≈99% of that mass doesn't come from the mass of the individual elementary particles but the strong force; potential energies of the quarks (and hence protons and neutrons) bound together can be translated to mass via E=mc^2. Another explanation for your magic could thus be not exactly one that works by meddling with gravity but meddling with another fundamental force which affects mass which in turn gets picked up unmodified gravity. The strong force is mediated by gluons (analogous to photons for the electromagnetic interactions), so off we go with gluemancy.



                                    The problem with this all is of course, that if one is capable enough to modify these tine, fundamental properties of particles, it doesn't easily make much sense to be able to only demonstrate gravity-based powers.






                                    share|improve this answer








                                    New contributor




                                    JoonasD6 is a new contributor to this site. Take care in asking for clarification, commenting, and answering.
                                    Check out our Code of Conduct.






                                    $endgroup$


















                                      0












                                      $begingroup$

                                      The Higgs field gives elementary particles their mass, and having some sort of a mechanism to attenuate or disable its interactions could work to your benefit at a level that doesn't really have a reasonably obvious counterargument. So in effect, the magic could somehow stop this interaction, making object momentarily more or less massive. (The remote pushing effect you mentioned I found the most difficult to explain...) The intricacies work very much differently for charged leptons and force carrier bosons, though.



                                      Most of the mass of matter is from the mass of the atomic nuclei, and ≈99% of that mass doesn't come from the mass of the individual elementary particles but the strong force; potential energies of the quarks (and hence protons and neutrons) bound together can be translated to mass via E=mc^2. Another explanation for your magic could thus be not exactly one that works by meddling with gravity but meddling with another fundamental force which affects mass which in turn gets picked up unmodified gravity. The strong force is mediated by gluons (analogous to photons for the electromagnetic interactions), so off we go with gluemancy.



                                      The problem with this all is of course, that if one is capable enough to modify these tine, fundamental properties of particles, it doesn't easily make much sense to be able to only demonstrate gravity-based powers.






                                      share|improve this answer








                                      New contributor




                                      JoonasD6 is a new contributor to this site. Take care in asking for clarification, commenting, and answering.
                                      Check out our Code of Conduct.






                                      $endgroup$
















                                        0












                                        0








                                        0





                                        $begingroup$

                                        The Higgs field gives elementary particles their mass, and having some sort of a mechanism to attenuate or disable its interactions could work to your benefit at a level that doesn't really have a reasonably obvious counterargument. So in effect, the magic could somehow stop this interaction, making object momentarily more or less massive. (The remote pushing effect you mentioned I found the most difficult to explain...) The intricacies work very much differently for charged leptons and force carrier bosons, though.



                                        Most of the mass of matter is from the mass of the atomic nuclei, and ≈99% of that mass doesn't come from the mass of the individual elementary particles but the strong force; potential energies of the quarks (and hence protons and neutrons) bound together can be translated to mass via E=mc^2. Another explanation for your magic could thus be not exactly one that works by meddling with gravity but meddling with another fundamental force which affects mass which in turn gets picked up unmodified gravity. The strong force is mediated by gluons (analogous to photons for the electromagnetic interactions), so off we go with gluemancy.



                                        The problem with this all is of course, that if one is capable enough to modify these tine, fundamental properties of particles, it doesn't easily make much sense to be able to only demonstrate gravity-based powers.






                                        share|improve this answer








                                        New contributor




                                        JoonasD6 is a new contributor to this site. Take care in asking for clarification, commenting, and answering.
                                        Check out our Code of Conduct.






                                        $endgroup$



                                        The Higgs field gives elementary particles their mass, and having some sort of a mechanism to attenuate or disable its interactions could work to your benefit at a level that doesn't really have a reasonably obvious counterargument. So in effect, the magic could somehow stop this interaction, making object momentarily more or less massive. (The remote pushing effect you mentioned I found the most difficult to explain...) The intricacies work very much differently for charged leptons and force carrier bosons, though.



                                        Most of the mass of matter is from the mass of the atomic nuclei, and ≈99% of that mass doesn't come from the mass of the individual elementary particles but the strong force; potential energies of the quarks (and hence protons and neutrons) bound together can be translated to mass via E=mc^2. Another explanation for your magic could thus be not exactly one that works by meddling with gravity but meddling with another fundamental force which affects mass which in turn gets picked up unmodified gravity. The strong force is mediated by gluons (analogous to photons for the electromagnetic interactions), so off we go with gluemancy.



                                        The problem with this all is of course, that if one is capable enough to modify these tine, fundamental properties of particles, it doesn't easily make much sense to be able to only demonstrate gravity-based powers.







                                        share|improve this answer








                                        New contributor




                                        JoonasD6 is a new contributor to this site. Take care in asking for clarification, commenting, and answering.
                                        Check out our Code of Conduct.









                                        share|improve this answer



                                        share|improve this answer






                                        New contributor




                                        JoonasD6 is a new contributor to this site. Take care in asking for clarification, commenting, and answering.
                                        Check out our Code of Conduct.









                                        answered 9 hours ago









                                        JoonasD6JoonasD6

                                        1




                                        1




                                        New contributor




                                        JoonasD6 is a new contributor to this site. Take care in asking for clarification, commenting, and answering.
                                        Check out our Code of Conduct.





                                        New contributor





                                        JoonasD6 is a new contributor to this site. Take care in asking for clarification, commenting, and answering.
                                        Check out our Code of Conduct.






                                        JoonasD6 is a new contributor to this site. Take care in asking for clarification, commenting, and answering.
                                        Check out our Code of Conduct.























                                            0












                                            $begingroup$

                                            The kind of magic you are talking about is anthropomorphic.





                                            • Slowing down yourself down when falling, so you don't hit the ground at full speed (as near to 0m/s as possible, at the very least you shouldn't be injured by landing) (e.g. jumping from higher up)

                                            • Levitating yourself / other objects, so they don't move in any direction

                                            • Bonus points: still won't move if influenced by another force (e.g. a thrown stone)

                                            • Pushing objects out of your way (e.g. a locked door)

                                            • Ability to combine multiple "gravity fields", so you could simultaneously keep a cup in place and rotate it upwards-down, while still keeping the liquid inside




                                            Gravity is the measurement of the degree to which mass-energy curves space time around it. What you describe above is a bunch of stuff that seems convincing to a human if you described gravity magic to.



                                            A physics based process won't neatly fit in a "stuff that X magic would do" that a human would narrate, because physics doesn't really care what a human finds narratively convincing.



                                            As an example, Electro-Magnitism covers everything from being able to touch things, see things, shoot lightning, and pretty much all of chemistry. Meanwhile, the human "story" based Magnatism is about being able to move metals around (because they are "magnetic"), and Electricty magic is about lightning bolts and static electricty.



                                            If you want anthropomorphic magic, magic that makes sense mythologically or in categories humans care about, it has to be powered by an intelligence with a similar mind to ours. That intelligence somehow set up the rules in ways that we find are pleasing.



                                            Spells become "instructions" to this intelligence, who then does what asked. The being being interacted with could be a god-like entity, a demon, or a post-singularity AI, or an ancient forgotten society of super-scientists who left "macros" active that their children used to play with.



                                            In the last case, the affinity of certain people for certain kinds of magic is akin to being able to unlock a certain smart phone with your voice, because it coincidentally is similar to the real owner (it doesn't have to be voice-locked, but you can imagine it being so). The fact that these ancient intelligences let their children play with fireballs would be explained by the fact that everyone in their society was backed up, and repair/resurrection was trivial; that technology doesn't recognize humans as being in their pattern-banks. Humans who can heal are actually using the "veternarian" macros.



                                            Now, once we divorce the physical mechanism from the "kind of magic", the fact that these are "gravity" macros is because they are the kind of thing that the parent programmed for their kid (or maybe bought the "play with gravity" kit for their kid's system).



                                            Your entire environment could be a simulation running in an ancient computer, with the original hosts long exinct. The matter you are walking on and breathing could be programmable computronium (mass-energy optimized to do computation) that emulates being matter so as to leave a "natural" experience for growing up, but being computronium this is just an option; "magic" literally ignores the apparent laws of physics, because the laws of physics as you experience them are just an emulation layer.



                                            Being able to float in either of these cases is little more than a matter of changing the position value of a variable in a specific coordinate space. Now, no human has access to this underlying system; so they cannot insert the wrong arguments and cause the water on the earth to start to orbit the other way around the sun.






                                            share|improve this answer









                                            $endgroup$


















                                              0












                                              $begingroup$

                                              The kind of magic you are talking about is anthropomorphic.





                                              • Slowing down yourself down when falling, so you don't hit the ground at full speed (as near to 0m/s as possible, at the very least you shouldn't be injured by landing) (e.g. jumping from higher up)

                                              • Levitating yourself / other objects, so they don't move in any direction

                                              • Bonus points: still won't move if influenced by another force (e.g. a thrown stone)

                                              • Pushing objects out of your way (e.g. a locked door)

                                              • Ability to combine multiple "gravity fields", so you could simultaneously keep a cup in place and rotate it upwards-down, while still keeping the liquid inside




                                              Gravity is the measurement of the degree to which mass-energy curves space time around it. What you describe above is a bunch of stuff that seems convincing to a human if you described gravity magic to.



                                              A physics based process won't neatly fit in a "stuff that X magic would do" that a human would narrate, because physics doesn't really care what a human finds narratively convincing.



                                              As an example, Electro-Magnitism covers everything from being able to touch things, see things, shoot lightning, and pretty much all of chemistry. Meanwhile, the human "story" based Magnatism is about being able to move metals around (because they are "magnetic"), and Electricty magic is about lightning bolts and static electricty.



                                              If you want anthropomorphic magic, magic that makes sense mythologically or in categories humans care about, it has to be powered by an intelligence with a similar mind to ours. That intelligence somehow set up the rules in ways that we find are pleasing.



                                              Spells become "instructions" to this intelligence, who then does what asked. The being being interacted with could be a god-like entity, a demon, or a post-singularity AI, or an ancient forgotten society of super-scientists who left "macros" active that their children used to play with.



                                              In the last case, the affinity of certain people for certain kinds of magic is akin to being able to unlock a certain smart phone with your voice, because it coincidentally is similar to the real owner (it doesn't have to be voice-locked, but you can imagine it being so). The fact that these ancient intelligences let their children play with fireballs would be explained by the fact that everyone in their society was backed up, and repair/resurrection was trivial; that technology doesn't recognize humans as being in their pattern-banks. Humans who can heal are actually using the "veternarian" macros.



                                              Now, once we divorce the physical mechanism from the "kind of magic", the fact that these are "gravity" macros is because they are the kind of thing that the parent programmed for their kid (or maybe bought the "play with gravity" kit for their kid's system).



                                              Your entire environment could be a simulation running in an ancient computer, with the original hosts long exinct. The matter you are walking on and breathing could be programmable computronium (mass-energy optimized to do computation) that emulates being matter so as to leave a "natural" experience for growing up, but being computronium this is just an option; "magic" literally ignores the apparent laws of physics, because the laws of physics as you experience them are just an emulation layer.



                                              Being able to float in either of these cases is little more than a matter of changing the position value of a variable in a specific coordinate space. Now, no human has access to this underlying system; so they cannot insert the wrong arguments and cause the water on the earth to start to orbit the other way around the sun.






                                              share|improve this answer









                                              $endgroup$
















                                                0












                                                0








                                                0





                                                $begingroup$

                                                The kind of magic you are talking about is anthropomorphic.





                                                • Slowing down yourself down when falling, so you don't hit the ground at full speed (as near to 0m/s as possible, at the very least you shouldn't be injured by landing) (e.g. jumping from higher up)

                                                • Levitating yourself / other objects, so they don't move in any direction

                                                • Bonus points: still won't move if influenced by another force (e.g. a thrown stone)

                                                • Pushing objects out of your way (e.g. a locked door)

                                                • Ability to combine multiple "gravity fields", so you could simultaneously keep a cup in place and rotate it upwards-down, while still keeping the liquid inside




                                                Gravity is the measurement of the degree to which mass-energy curves space time around it. What you describe above is a bunch of stuff that seems convincing to a human if you described gravity magic to.



                                                A physics based process won't neatly fit in a "stuff that X magic would do" that a human would narrate, because physics doesn't really care what a human finds narratively convincing.



                                                As an example, Electro-Magnitism covers everything from being able to touch things, see things, shoot lightning, and pretty much all of chemistry. Meanwhile, the human "story" based Magnatism is about being able to move metals around (because they are "magnetic"), and Electricty magic is about lightning bolts and static electricty.



                                                If you want anthropomorphic magic, magic that makes sense mythologically or in categories humans care about, it has to be powered by an intelligence with a similar mind to ours. That intelligence somehow set up the rules in ways that we find are pleasing.



                                                Spells become "instructions" to this intelligence, who then does what asked. The being being interacted with could be a god-like entity, a demon, or a post-singularity AI, or an ancient forgotten society of super-scientists who left "macros" active that their children used to play with.



                                                In the last case, the affinity of certain people for certain kinds of magic is akin to being able to unlock a certain smart phone with your voice, because it coincidentally is similar to the real owner (it doesn't have to be voice-locked, but you can imagine it being so). The fact that these ancient intelligences let their children play with fireballs would be explained by the fact that everyone in their society was backed up, and repair/resurrection was trivial; that technology doesn't recognize humans as being in their pattern-banks. Humans who can heal are actually using the "veternarian" macros.



                                                Now, once we divorce the physical mechanism from the "kind of magic", the fact that these are "gravity" macros is because they are the kind of thing that the parent programmed for their kid (or maybe bought the "play with gravity" kit for their kid's system).



                                                Your entire environment could be a simulation running in an ancient computer, with the original hosts long exinct. The matter you are walking on and breathing could be programmable computronium (mass-energy optimized to do computation) that emulates being matter so as to leave a "natural" experience for growing up, but being computronium this is just an option; "magic" literally ignores the apparent laws of physics, because the laws of physics as you experience them are just an emulation layer.



                                                Being able to float in either of these cases is little more than a matter of changing the position value of a variable in a specific coordinate space. Now, no human has access to this underlying system; so they cannot insert the wrong arguments and cause the water on the earth to start to orbit the other way around the sun.






                                                share|improve this answer









                                                $endgroup$



                                                The kind of magic you are talking about is anthropomorphic.





                                                • Slowing down yourself down when falling, so you don't hit the ground at full speed (as near to 0m/s as possible, at the very least you shouldn't be injured by landing) (e.g. jumping from higher up)

                                                • Levitating yourself / other objects, so they don't move in any direction

                                                • Bonus points: still won't move if influenced by another force (e.g. a thrown stone)

                                                • Pushing objects out of your way (e.g. a locked door)

                                                • Ability to combine multiple "gravity fields", so you could simultaneously keep a cup in place and rotate it upwards-down, while still keeping the liquid inside




                                                Gravity is the measurement of the degree to which mass-energy curves space time around it. What you describe above is a bunch of stuff that seems convincing to a human if you described gravity magic to.



                                                A physics based process won't neatly fit in a "stuff that X magic would do" that a human would narrate, because physics doesn't really care what a human finds narratively convincing.



                                                As an example, Electro-Magnitism covers everything from being able to touch things, see things, shoot lightning, and pretty much all of chemistry. Meanwhile, the human "story" based Magnatism is about being able to move metals around (because they are "magnetic"), and Electricty magic is about lightning bolts and static electricty.



                                                If you want anthropomorphic magic, magic that makes sense mythologically or in categories humans care about, it has to be powered by an intelligence with a similar mind to ours. That intelligence somehow set up the rules in ways that we find are pleasing.



                                                Spells become "instructions" to this intelligence, who then does what asked. The being being interacted with could be a god-like entity, a demon, or a post-singularity AI, or an ancient forgotten society of super-scientists who left "macros" active that their children used to play with.



                                                In the last case, the affinity of certain people for certain kinds of magic is akin to being able to unlock a certain smart phone with your voice, because it coincidentally is similar to the real owner (it doesn't have to be voice-locked, but you can imagine it being so). The fact that these ancient intelligences let their children play with fireballs would be explained by the fact that everyone in their society was backed up, and repair/resurrection was trivial; that technology doesn't recognize humans as being in their pattern-banks. Humans who can heal are actually using the "veternarian" macros.



                                                Now, once we divorce the physical mechanism from the "kind of magic", the fact that these are "gravity" macros is because they are the kind of thing that the parent programmed for their kid (or maybe bought the "play with gravity" kit for their kid's system).



                                                Your entire environment could be a simulation running in an ancient computer, with the original hosts long exinct. The matter you are walking on and breathing could be programmable computronium (mass-energy optimized to do computation) that emulates being matter so as to leave a "natural" experience for growing up, but being computronium this is just an option; "magic" literally ignores the apparent laws of physics, because the laws of physics as you experience them are just an emulation layer.



                                                Being able to float in either of these cases is little more than a matter of changing the position value of a variable in a specific coordinate space. Now, no human has access to this underlying system; so they cannot insert the wrong arguments and cause the water on the earth to start to orbit the other way around the sun.







                                                share|improve this answer












                                                share|improve this answer



                                                share|improve this answer










                                                answered 5 hours ago









                                                YakkYakk

                                                8,81911238




                                                8,81911238






























                                                    draft saved

                                                    draft discarded




















































                                                    Thanks for contributing an answer to Worldbuilding Stack Exchange!


                                                    • Please be sure to answer the question. Provide details and share your research!

                                                    But avoid



                                                    • Asking for help, clarification, or responding to other answers.

                                                    • Making statements based on opinion; back them up with references or personal experience.


                                                    Use MathJax to format equations. MathJax reference.


                                                    To learn more, see our tips on writing great answers.




                                                    draft saved


                                                    draft discarded














                                                    StackExchange.ready(
                                                    function () {
                                                    StackExchange.openid.initPostLogin('.new-post-login', 'https%3a%2f%2fworldbuilding.stackexchange.com%2fquestions%2f141533%2fgravity-magic-how-does-it-work%23new-answer', 'question_page');
                                                    }
                                                    );

                                                    Post as a guest















                                                    Required, but never shown





















































                                                    Required, but never shown














                                                    Required, but never shown












                                                    Required, but never shown







                                                    Required, but never shown

































                                                    Required, but never shown














                                                    Required, but never shown












                                                    Required, but never shown







                                                    Required, but never shown







                                                    Popular posts from this blog

                                                    How did Captain America manage to do this?

                                                    迪纳利

                                                    南乌拉尔铁路局